Use \dd for derivative d in Y&F-12 28.12 and 30
[course.git] / latex / problems / Serway_and_Jewett_4_venkat / problem27.V1.tex
1 \begin{problem}
2 A double slit interference pattern is produced by two slits each of
3 width $0.35\U{mm}$ and slit separation $4.2\U{mm}$ on a screen placed
4 parallel to the slits at a distance $1.5\U{m}$ from the slits. The
5 slits are illuminated by light of wavelength $580\U{nm}$. Given:
6 $Y_m=m(D\lambda/d)$ \Part{a} What is the angular separation (in
7 degrees) between the central fringe and the 8th bright
8 fringe? \Part{b} What is the fringe width (in $\mu$m) of the pattern
9 on the screen? \Part{c} Assuming that we can see a large number of
10 fringes on the screen, which bright fringes between the central fringe
11 and the 40th bright fringe will be missing on the screen? Show
12 appropriate calculations and give reasons to substantiate your
13 answer. \Part{d} If however, a thin mica sheet of index of refraction
14 $n=1.58$ and thickness $t$ covers one of the slits, the central point
15 on the screen is now occupied by the $29th$ bright fringe. What is the
16 thickness $t$ (in $\mu$m) of the mica sheet?
17 \end{problem}
18
19 \begin{solution}
20 \Part{a}
21 Bright fringes appear when the pathlengths differ by integer numbers
22 of wavelengths.
23 \begin{align}
24   d\sin(\theta) &= m \lambda \label{eq.doubleslit_max_angle} \\
25   \theta &= \arcsin\p({\frac{m\lambda}{d}}) \\
26   \theta_0 &= \arcsin(0) = 0\dg \\
27   \theta_8 &= \arcsin\p({\frac{8\lambda}{d}})
28     = \arcsin\p({\frac{8\cdot580\E{-9}\U{m}}{4.2\E{-3}\U{m}}})
29     = 0.0633\dg \\
30   \Delta \theta &= \theta_8 - \theta_0 = \ans{0.0633\dg}
31 \end{align}
32
33 \Part{b}
34 To find the fringe width we take the small-angle approximation of
35 Eqn.~\ref{eq.doubleslit_max_angle}
36 \begin{align}
37   Y_m &= D\tan(\theta) \approx \frac{D}{d}d\sin(\theta)
38     = \frac{D}{d} m\lambda \\
39   \Delta y &= y_1 - y_0 = \frac{D\lambda}{d}
40     = \frac{1.5\U{m}\cdot580\E{-9}\U{m}}{4.2\E{-3}\U{m}}
41     = \ans{207\U{$\mu$m}}
42 \end{align}
43
44 \Part{c}
45 Up to now we have ignored the larger envelope caused by the
46 single-slit interference inside each slit.  That has interference
47 minima when the slit can be broken up into an integer number of
48 self-annihilating regions.  The difference in path length between rays
49 from the top and bottom of a self-annihilating region should be
50 $\lambda$ (so that the ray from the top cancels the ray from the
51 middle, the ray slightly below the top cancels the ray slightly below
52 the middle, etc.)  With this idea and a little trig we find length
53 along the slit of a self-annihilating region should be given by
54 \begin{align}
55  x\sin(\theta) &= \lambda  &
56  x &= \frac{\lambda}{\sin(\theta)}
57 \end{align}
58 So there are interference minima when we can fit an integer number of
59 these regions into $a$:
60 \begin{align}
61   a &= nx = n \frac{\lambda}{\sin(\theta)} \\
62   a\sin(\theta) &= n\lambda
63 \end{align}
64 for any non-zero, integer $n$.  The single-slit interference effect
65 kills a double-slit maximum when they occur at the same angle
66 \begin{align}
67   \sin(\theta) &= \frac{n}{a}\lambda = \frac{m}{d}\lambda \\
68   \frac{n}{a} &= \frac{m}{d} \\
69   \frac{m}{n} &= \frac{d}{a} = \frac{4.2\U{mm}}{0.35\U{mm}} = 12
70 \end{align}
71 So the $m(n=1)=\ans{12^\text{th}}$, $m(n=2)=\ans{24^\text{th}}$, and
72 $m(n=3)=\ans{36^\text{th}}$ fringes are missing.
73
74 \Part{d}
75 While passing through the mica (almost perpendicularly), the one ray
76 travels an `extra' $nt-t=(n-1)t$ wavelengths (compared to it's
77 neighbor passing through air).  Because the central point on the
78 screen is the $29^\text{th}$ fringe, and the $0^\text{th}$ fringe
79 occurs where the effective path lengths are equal, the mica-ray must
80 have traveled an extra $29\lambda$ of effective distance.  Therefore
81 \begin{align}
82   29\lambda &= (n-1)t \\
83   t &= \frac{29\lambda}{n-1}
84     = \frac{29\cdot580\E{-9}\U{m}}{0.58} = \ans{29.0\U{$\mu$m}}
85 \end{align}
86
87 By `effective' distances, I mean `measured in wavelengths traveled',
88 as opposed to `measured in meters'.
89 \end{solution}